Apostol Calculus, Method of Exhaustion











up vote
0
down vote

favorite












In Apostol's Calculus, he goes through the method of exhaustion to find the area under a parabola from $0 to b$. Using the fact that,



begin{align}
&1^2+2^2+...+(n-1)^2 < frac{n^3}{3} < 1^2+2^2+...+n^2label{1} \
&Rightarrow frac{b^3}{n^3}(1^2+2^2+...+(n-1)^2) < frac{b^3}{3} < frac{b^3}{n^3}(1^2+2^2+...+n^2) nonumber \
&Rightarrow s_{n} < frac{b^3}{3} < S_{n} nonumber
end{align}



where $s_{n}$ and $S_{n}$ are the lower and upper approximation (rectangles), respectively, for the area under the parabola. We then have to show that $A=frac{b^3}{3}$ is the only number that satisfies



begin{equation}
s_{n}<A<S_{n} label{2}
end{equation}



for every $ngeq1$. Using the left-most side of the first inequality, he adds $n^2$ and then multiplies both sides by $frac{b^3}{n^3}$,



begin{align*}
&frac{b^3}{n^3}(1^2+2^2+...+n^2)<frac{b^3}{3}+frac{b^3}{n^2} \
&Rightarrow S_{n}<frac{b^3}{3}+frac{b^3}{n^2}
end{align*}



for the right-most side of the inequality, he subtracts $n^2$ and multiplies by $frac{b^3}{n^3}$,



begin{align*}
&frac{b^3}{3}-frac{b^3}{n^2}<frac{b^3}{n^3}(1^2+2^2+...+(n-1)^2)\
&Rightarrow frac{b^3}{3}-frac{b^3}{n^3}<s_{n}
end{align*}



this implies,



begin{align*}
frac{b^3}{3}-frac{b^3}{n^2}<A<frac{b^3}{3}+frac{b^3}{n^2}
end{align*}



This is where it gets confusing. He says the only possibilities are:



$$begin{array}{ccc}
A>frac{b^3}{3},& A<frac{b^3}{3},& A=frac{b^3}{3}
end{array}$$



and proceeds to show that $A=frac{b^3}{3}$ via contradictions for the first two cases. This is fine, but what about $frac{b^3}{n^2}$ in the inequality? why don't we have to consider the possible relationships between $A$ and $frac{b^3}{n^2}$?










share|cite|improve this question
























  • You have a confusing typo. The far LHS and far RHS of the inequality above "This is where it get confusing" are identical. Please edit.
    – DanielWainfleet
    yesterday










  • I see! I have fixed it, thank you very much.
    – Jonathan Duran
    yesterday






  • 1




    For $bne 0$ we have $|b^{-3}(A-b^3/3|<1/n^2$ for $all$ $nin Bbb Z^+$, which is not possible unless $b^{-3}(A-b^3/3)=0$, which is not possible unless $A=b^3/3$. Note that $A$ and $b$ have no relation to $n$.
    – DanielWainfleet
    yesterday












  • What about $A<frac{b^3}{3}$, wouldn't that satisfy the inequality in your response?
    – Jonathan Duran
    14 hours ago















up vote
0
down vote

favorite












In Apostol's Calculus, he goes through the method of exhaustion to find the area under a parabola from $0 to b$. Using the fact that,



begin{align}
&1^2+2^2+...+(n-1)^2 < frac{n^3}{3} < 1^2+2^2+...+n^2label{1} \
&Rightarrow frac{b^3}{n^3}(1^2+2^2+...+(n-1)^2) < frac{b^3}{3} < frac{b^3}{n^3}(1^2+2^2+...+n^2) nonumber \
&Rightarrow s_{n} < frac{b^3}{3} < S_{n} nonumber
end{align}



where $s_{n}$ and $S_{n}$ are the lower and upper approximation (rectangles), respectively, for the area under the parabola. We then have to show that $A=frac{b^3}{3}$ is the only number that satisfies



begin{equation}
s_{n}<A<S_{n} label{2}
end{equation}



for every $ngeq1$. Using the left-most side of the first inequality, he adds $n^2$ and then multiplies both sides by $frac{b^3}{n^3}$,



begin{align*}
&frac{b^3}{n^3}(1^2+2^2+...+n^2)<frac{b^3}{3}+frac{b^3}{n^2} \
&Rightarrow S_{n}<frac{b^3}{3}+frac{b^3}{n^2}
end{align*}



for the right-most side of the inequality, he subtracts $n^2$ and multiplies by $frac{b^3}{n^3}$,



begin{align*}
&frac{b^3}{3}-frac{b^3}{n^2}<frac{b^3}{n^3}(1^2+2^2+...+(n-1)^2)\
&Rightarrow frac{b^3}{3}-frac{b^3}{n^3}<s_{n}
end{align*}



this implies,



begin{align*}
frac{b^3}{3}-frac{b^3}{n^2}<A<frac{b^3}{3}+frac{b^3}{n^2}
end{align*}



This is where it gets confusing. He says the only possibilities are:



$$begin{array}{ccc}
A>frac{b^3}{3},& A<frac{b^3}{3},& A=frac{b^3}{3}
end{array}$$



and proceeds to show that $A=frac{b^3}{3}$ via contradictions for the first two cases. This is fine, but what about $frac{b^3}{n^2}$ in the inequality? why don't we have to consider the possible relationships between $A$ and $frac{b^3}{n^2}$?










share|cite|improve this question
























  • You have a confusing typo. The far LHS and far RHS of the inequality above "This is where it get confusing" are identical. Please edit.
    – DanielWainfleet
    yesterday










  • I see! I have fixed it, thank you very much.
    – Jonathan Duran
    yesterday






  • 1




    For $bne 0$ we have $|b^{-3}(A-b^3/3|<1/n^2$ for $all$ $nin Bbb Z^+$, which is not possible unless $b^{-3}(A-b^3/3)=0$, which is not possible unless $A=b^3/3$. Note that $A$ and $b$ have no relation to $n$.
    – DanielWainfleet
    yesterday












  • What about $A<frac{b^3}{3}$, wouldn't that satisfy the inequality in your response?
    – Jonathan Duran
    14 hours ago













up vote
0
down vote

favorite









up vote
0
down vote

favorite











In Apostol's Calculus, he goes through the method of exhaustion to find the area under a parabola from $0 to b$. Using the fact that,



begin{align}
&1^2+2^2+...+(n-1)^2 < frac{n^3}{3} < 1^2+2^2+...+n^2label{1} \
&Rightarrow frac{b^3}{n^3}(1^2+2^2+...+(n-1)^2) < frac{b^3}{3} < frac{b^3}{n^3}(1^2+2^2+...+n^2) nonumber \
&Rightarrow s_{n} < frac{b^3}{3} < S_{n} nonumber
end{align}



where $s_{n}$ and $S_{n}$ are the lower and upper approximation (rectangles), respectively, for the area under the parabola. We then have to show that $A=frac{b^3}{3}$ is the only number that satisfies



begin{equation}
s_{n}<A<S_{n} label{2}
end{equation}



for every $ngeq1$. Using the left-most side of the first inequality, he adds $n^2$ and then multiplies both sides by $frac{b^3}{n^3}$,



begin{align*}
&frac{b^3}{n^3}(1^2+2^2+...+n^2)<frac{b^3}{3}+frac{b^3}{n^2} \
&Rightarrow S_{n}<frac{b^3}{3}+frac{b^3}{n^2}
end{align*}



for the right-most side of the inequality, he subtracts $n^2$ and multiplies by $frac{b^3}{n^3}$,



begin{align*}
&frac{b^3}{3}-frac{b^3}{n^2}<frac{b^3}{n^3}(1^2+2^2+...+(n-1)^2)\
&Rightarrow frac{b^3}{3}-frac{b^3}{n^3}<s_{n}
end{align*}



this implies,



begin{align*}
frac{b^3}{3}-frac{b^3}{n^2}<A<frac{b^3}{3}+frac{b^3}{n^2}
end{align*}



This is where it gets confusing. He says the only possibilities are:



$$begin{array}{ccc}
A>frac{b^3}{3},& A<frac{b^3}{3},& A=frac{b^3}{3}
end{array}$$



and proceeds to show that $A=frac{b^3}{3}$ via contradictions for the first two cases. This is fine, but what about $frac{b^3}{n^2}$ in the inequality? why don't we have to consider the possible relationships between $A$ and $frac{b^3}{n^2}$?










share|cite|improve this question















In Apostol's Calculus, he goes through the method of exhaustion to find the area under a parabola from $0 to b$. Using the fact that,



begin{align}
&1^2+2^2+...+(n-1)^2 < frac{n^3}{3} < 1^2+2^2+...+n^2label{1} \
&Rightarrow frac{b^3}{n^3}(1^2+2^2+...+(n-1)^2) < frac{b^3}{3} < frac{b^3}{n^3}(1^2+2^2+...+n^2) nonumber \
&Rightarrow s_{n} < frac{b^3}{3} < S_{n} nonumber
end{align}



where $s_{n}$ and $S_{n}$ are the lower and upper approximation (rectangles), respectively, for the area under the parabola. We then have to show that $A=frac{b^3}{3}$ is the only number that satisfies



begin{equation}
s_{n}<A<S_{n} label{2}
end{equation}



for every $ngeq1$. Using the left-most side of the first inequality, he adds $n^2$ and then multiplies both sides by $frac{b^3}{n^3}$,



begin{align*}
&frac{b^3}{n^3}(1^2+2^2+...+n^2)<frac{b^3}{3}+frac{b^3}{n^2} \
&Rightarrow S_{n}<frac{b^3}{3}+frac{b^3}{n^2}
end{align*}



for the right-most side of the inequality, he subtracts $n^2$ and multiplies by $frac{b^3}{n^3}$,



begin{align*}
&frac{b^3}{3}-frac{b^3}{n^2}<frac{b^3}{n^3}(1^2+2^2+...+(n-1)^2)\
&Rightarrow frac{b^3}{3}-frac{b^3}{n^3}<s_{n}
end{align*}



this implies,



begin{align*}
frac{b^3}{3}-frac{b^3}{n^2}<A<frac{b^3}{3}+frac{b^3}{n^2}
end{align*}



This is where it gets confusing. He says the only possibilities are:



$$begin{array}{ccc}
A>frac{b^3}{3},& A<frac{b^3}{3},& A=frac{b^3}{3}
end{array}$$



and proceeds to show that $A=frac{b^3}{3}$ via contradictions for the first two cases. This is fine, but what about $frac{b^3}{n^2}$ in the inequality? why don't we have to consider the possible relationships between $A$ and $frac{b^3}{n^2}$?







calculus area






share|cite|improve this question















share|cite|improve this question













share|cite|improve this question




share|cite|improve this question








edited yesterday

























asked 2 days ago









Jonathan Duran

504




504












  • You have a confusing typo. The far LHS and far RHS of the inequality above "This is where it get confusing" are identical. Please edit.
    – DanielWainfleet
    yesterday










  • I see! I have fixed it, thank you very much.
    – Jonathan Duran
    yesterday






  • 1




    For $bne 0$ we have $|b^{-3}(A-b^3/3|<1/n^2$ for $all$ $nin Bbb Z^+$, which is not possible unless $b^{-3}(A-b^3/3)=0$, which is not possible unless $A=b^3/3$. Note that $A$ and $b$ have no relation to $n$.
    – DanielWainfleet
    yesterday












  • What about $A<frac{b^3}{3}$, wouldn't that satisfy the inequality in your response?
    – Jonathan Duran
    14 hours ago


















  • You have a confusing typo. The far LHS and far RHS of the inequality above "This is where it get confusing" are identical. Please edit.
    – DanielWainfleet
    yesterday










  • I see! I have fixed it, thank you very much.
    – Jonathan Duran
    yesterday






  • 1




    For $bne 0$ we have $|b^{-3}(A-b^3/3|<1/n^2$ for $all$ $nin Bbb Z^+$, which is not possible unless $b^{-3}(A-b^3/3)=0$, which is not possible unless $A=b^3/3$. Note that $A$ and $b$ have no relation to $n$.
    – DanielWainfleet
    yesterday












  • What about $A<frac{b^3}{3}$, wouldn't that satisfy the inequality in your response?
    – Jonathan Duran
    14 hours ago
















You have a confusing typo. The far LHS and far RHS of the inequality above "This is where it get confusing" are identical. Please edit.
– DanielWainfleet
yesterday




You have a confusing typo. The far LHS and far RHS of the inequality above "This is where it get confusing" are identical. Please edit.
– DanielWainfleet
yesterday












I see! I have fixed it, thank you very much.
– Jonathan Duran
yesterday




I see! I have fixed it, thank you very much.
– Jonathan Duran
yesterday




1




1




For $bne 0$ we have $|b^{-3}(A-b^3/3|<1/n^2$ for $all$ $nin Bbb Z^+$, which is not possible unless $b^{-3}(A-b^3/3)=0$, which is not possible unless $A=b^3/3$. Note that $A$ and $b$ have no relation to $n$.
– DanielWainfleet
yesterday






For $bne 0$ we have $|b^{-3}(A-b^3/3|<1/n^2$ for $all$ $nin Bbb Z^+$, which is not possible unless $b^{-3}(A-b^3/3)=0$, which is not possible unless $A=b^3/3$. Note that $A$ and $b$ have no relation to $n$.
– DanielWainfleet
yesterday














What about $A<frac{b^3}{3}$, wouldn't that satisfy the inequality in your response?
– Jonathan Duran
14 hours ago




What about $A<frac{b^3}{3}$, wouldn't that satisfy the inequality in your response?
– Jonathan Duran
14 hours ago















active

oldest

votes











Your Answer





StackExchange.ifUsing("editor", function () {
return StackExchange.using("mathjaxEditing", function () {
StackExchange.MarkdownEditor.creationCallbacks.add(function (editor, postfix) {
StackExchange.mathjaxEditing.prepareWmdForMathJax(editor, postfix, [["$", "$"], ["\\(","\\)"]]);
});
});
}, "mathjax-editing");

StackExchange.ready(function() {
var channelOptions = {
tags: "".split(" "),
id: "69"
};
initTagRenderer("".split(" "), "".split(" "), channelOptions);

StackExchange.using("externalEditor", function() {
// Have to fire editor after snippets, if snippets enabled
if (StackExchange.settings.snippets.snippetsEnabled) {
StackExchange.using("snippets", function() {
createEditor();
});
}
else {
createEditor();
}
});

function createEditor() {
StackExchange.prepareEditor({
heartbeatType: 'answer',
convertImagesToLinks: true,
noModals: true,
showLowRepImageUploadWarning: true,
reputationToPostImages: 10,
bindNavPrevention: true,
postfix: "",
imageUploader: {
brandingHtml: "Powered by u003ca class="icon-imgur-white" href="https://imgur.com/"u003eu003c/au003e",
contentPolicyHtml: "User contributions licensed under u003ca href="https://creativecommons.org/licenses/by-sa/3.0/"u003ecc by-sa 3.0 with attribution requiredu003c/au003e u003ca href="https://stackoverflow.com/legal/content-policy"u003e(content policy)u003c/au003e",
allowUrls: true
},
noCode: true, onDemand: true,
discardSelector: ".discard-answer"
,immediatelyShowMarkdownHelp:true
});


}
});














 

draft saved


draft discarded


















StackExchange.ready(
function () {
StackExchange.openid.initPostLogin('.new-post-login', 'https%3a%2f%2fmath.stackexchange.com%2fquestions%2f2997831%2fapostol-calculus-method-of-exhaustion%23new-answer', 'question_page');
}
);

Post as a guest















Required, but never shown






























active

oldest

votes













active

oldest

votes









active

oldest

votes






active

oldest

votes
















 

draft saved


draft discarded



















































 


draft saved


draft discarded














StackExchange.ready(
function () {
StackExchange.openid.initPostLogin('.new-post-login', 'https%3a%2f%2fmath.stackexchange.com%2fquestions%2f2997831%2fapostol-calculus-method-of-exhaustion%23new-answer', 'question_page');
}
);

Post as a guest















Required, but never shown





















































Required, but never shown














Required, but never shown












Required, but never shown







Required, but never shown

































Required, but never shown














Required, but never shown












Required, but never shown







Required, but never shown







Popular posts from this blog

Plaza Victoria

In PowerPoint, is there a keyboard shortcut for bulleted / numbered list?

How to put 3 figures in Latex with 2 figures side by side and 1 below these side by side images but in...